Difference between revisions of "2017 AMC 8 Problems/Problem 4"

(Created page with "==Problem 4== When <math>0.000315</math> is multiplied by <math>7,928,564</math> the product is closest to which of the following? <math>\textbf{(A) }210\qquad\textbf{(B) }24...")
 
(8 intermediate revisions by 5 users not shown)
Line 1: Line 1:
==Problem 4==
+
==Problem==
 
When <math>0.000315</math> is multiplied by <math>7,928,564</math> the product is closest to which of the following?
 
When <math>0.000315</math> is multiplied by <math>7,928,564</math> the product is closest to which of the following?
  
 
<math>\textbf{(A) }210\qquad\textbf{(B) }240\qquad\textbf{(C) }2100\qquad\textbf{(D) }2400\qquad\textbf{(E) }24000</math>
 
<math>\textbf{(A) }210\qquad\textbf{(B) }240\qquad\textbf{(C) }2100\qquad\textbf{(D) }2400\qquad\textbf{(E) }24000</math>
  
==Solution:==
+
==Solution==
We can approximate <math>7,928,564</math> to <math>8,000,000,</math> and <math>0.000315</math> to <math>0.0003.</math> Multiplying the two yields <math>2400.</math> This gives our answer to be <math>\text{D)}2400.</math>
+
We can approximate <math>7,928,564</math> to <math>8,000,000,</math> and <math>0.000315</math> to <math>0.0003.</math> Multiplying the two yields <math>2400.</math> Thus shows our answer is <math>\boxed{\textbf{(D)}\ 2400}.</math>
 +
 
 +
==Video Solution==
 +
https://youtu.be/cY4NYSAD0vQ
 +
 
 +
==See Also==
 +
{{AMC8 box|year=2017|num-b=3|num-a=5}}
 +
 
 +
{{MAA Notice}}

Revision as of 13:59, 18 January 2021

Problem

When $0.000315$ is multiplied by $7,928,564$ the product is closest to which of the following?

$\textbf{(A) }210\qquad\textbf{(B) }240\qquad\textbf{(C) }2100\qquad\textbf{(D) }2400\qquad\textbf{(E) }24000$

Solution

We can approximate $7,928,564$ to $8,000,000,$ and $0.000315$ to $0.0003.$ Multiplying the two yields $2400.$ Thus shows our answer is $\boxed{\textbf{(D)}\ 2400}.$

Video Solution

https://youtu.be/cY4NYSAD0vQ

See Also

2017 AMC 8 (ProblemsAnswer KeyResources)
Preceded by
Problem 3
Followed by
Problem 5
1 2 3 4 5 6 7 8 9 10 11 12 13 14 15 16 17 18 19 20 21 22 23 24 25
All AJHSME/AMC 8 Problems and Solutions

The problems on this page are copyrighted by the Mathematical Association of America's American Mathematics Competitions. AMC logo.png